Đến nội dung

xuanhoan23112002 nội dung

Có 95 mục bởi xuanhoan23112002 (Tìm giới hạn từ 29-04-2020)



Sắp theo                Sắp xếp  

#709807 Bất đẳng thức

Đã gửi bởi xuanhoan23112002 on 03-06-2018 - 07:54 trong Bất đẳng thức - Cực trị

Cho a, b, c là các số thực không âm. Chứng minh rằng:

$3(a+b+c)\geq 2(\sqrt{a^2+bc}+\sqrt{b^2+ca}+\sqrt{c^2+ab})$




#711677 $f(x)=ax^4+bx+c> 0 \forall x> 0$

Đã gửi bởi xuanhoan23112002 on 27-06-2018 - 16:55 trong Đa thức

Cho $a\neq 0$ và $f(x)=ax^4+bx+c> 0 \forall x> 0$

CMR: $f(x)$ được biểu diễn ở dạng tổng bình phương của 2 tam thức bậc hai.




#707185 Tìm giá trị nhỏ nhất của: $S=a_{1}^{2}+a_{2...

Đã gửi bởi xuanhoan23112002 on 28-04-2018 - 15:19 trong Bất đẳng thức - Cực trị

Problem: Cho $a_{1}, a_{2},...,a_{19}$ là các số tự nhiên thỏa mãn: $a_{1}+a_{2}+...+a_{19}=26.$ Tìm giá trị nhỏ nhất của: $S=a_{1}^{2}+a_{2}^{2}+...+a_{19}^{2}.$

 




#709479 Bài tập về đa thức

Đã gửi bởi xuanhoan23112002 on 29-05-2018 - 08:53 trong Đa thức

Bài 1: Cho đa thức $f(x)=x^{2018}+\sum a_ix^{i}($a_i\in {-1,1}, $\forall i\in \left \{ 0,1,...,2017 \right \}$$)$ không có nghiệm thực. Tìm số lớn nhất các hệ số = -1 trong f(x)

Bài 2: Tìm đa thức P(x) hệ số thực thỏa mãn:

$(P(x))^{3}-3(P(x))^{2}=P(x^{3})-3P(-x)$, với mọi x là số thực




#709566 Số chính phương

Đã gửi bởi xuanhoan23112002 on 30-05-2018 - 09:39 trong Số học

Bài toán này sử dụng phương pháp bước nhảy Viete. Các bài viết khác về bước nhảy Viete trên VMF

http://diendantoanho...ước-nhảy-viete/

Lời giải của bài toán trên bạn có thể tham khảo ở đây: http://math.stackexc...-its-an-integer




#709553 số học

Đã gửi bởi xuanhoan23112002 on 30-05-2018 - 07:32 trong Số học

Từ giả thiết ta thấy ngay a, b, c đều là các số lẻ mà một số chính phương lẻ chia 8 dư 1

Từ nhận xét trên: $a^{30}+b^{4}+c^{2018}\equiv 3$ (mod 8)




#710109 Đề thi tuyển sinh vào lớp 10 THPT tỉnh Bắc Giang năm học 2018-2019

Đã gửi bởi xuanhoan23112002 on 06-06-2018 - 15:07 trong Tài liệu - Đề thi

Câu 5: Ta có:

$P=\frac{81x^2+18225x+1}{9x}-\frac{6\sqrt{x}+8}{x+1}\geq \frac{18x}{9x}-\frac{9x+9}{x+1}+2025= 2018$ (bất đẳng thức Cauchy)

Đẳng thức xảy ra $\Leftrightarrow x=\frac{1}{9}> 0$

Vậy $MinP=2018\Leftrightarrow x=\frac{1}{9}$




#710198 AP vuông góc với IJ

Đã gửi bởi xuanhoan23112002 on 07-06-2018 - 14:17 trong Hình học

Cho tam giác ABC với AC > AB. Các đường cao BB, CCcủa tam giác cắt nhau tại H. Gọi M, N lần lượt là trung điểm của BC', CB'. MH cắt đường tròn ngoại tiếp tam giác CHB' tại I; N

H cắt đường tròn ngoại tiếp tam giác BHC' tại J. Giả sử P là trung điểm cạnh BC. Chứng minh: AP vuông góc với IJ




#710187 Tâm đường tròn ngoại tiếp tứ giác ABCD nằm trên SI

Đã gửi bởi xuanhoan23112002 on 07-06-2018 - 11:51 trong Hình học

Cho tứ giác ABCD ngoại tiếp đường tròn tâm I. Giả sử bên trong tứ giác ta vẽ được 4 đường tròn bằng nhau và cùng đi qua 1 điểm S, và mỗi đường tròn tiếp xúc với 2 cạnh liên tiếp của tứ giác đó. Chứng minh tứ giác ABCD nội tiếp 1 đường tròn và tâm đường tròn đó nằm trên SI.




#707928 Bất đẳng thức

Đã gửi bởi xuanhoan23112002 on 08-05-2018 - 21:35 trong Bất đẳng thức và cực trị

BĐT$\Leftrightarrow \frac{a^2}{ab+ac}+\frac{b^2}{bc+bd}+\frac{c^2}{cd+ca}+\frac{d^2}{da+db}\geq 2$

Ta có VT$\geq \frac{(a+b+c+d)^2}{(a+d)(b+c)+(c+d)(a+b)}$( theo BĐT Cauchy-Schwarz)

Mà cũng theo BĐT AM-GM ta cũng có $(a+d)(b+c)+(a+b)(c+d)\leq \frac{(a+b+c+d)^2}{2}$

Do đó VT$\geq 2$

Vậy bất đẳng thức được chứng minh. Đẳng thức xảy ra$\Leftrightarrow$ $a= b= c= d> 0$




#705136 Chứng minh rằng với mọi số thực dương $a,b>2$ thì $2^a-1...

Đã gửi bởi xuanhoan23112002 on 06-04-2018 - 22:59 trong Số học

Bài toán sai khi a chia hết cho b 

Nếu a không chia hết cho b. Đặt a=bq+r(0<r<b)

Sử dụng phản chứng để suy ra 2r - 1 chia hết cho 2b - 1 (điều này vô lí do 0<2r - 1<2b - 1)

Từ đó ta có điều phải chứng minh.




#705630 Số học

Đã gửi bởi xuanhoan23112002 on 12-04-2018 - 20:09 trong Số học

Tìm các số nguyên dương n sao cho: Với mọi a, b là các số nguyên dương, nếu a2b+1 chia hết cho n thì a2+b cũng chia hết cho n




#704920 Lập phương trình đt d qua M

Đã gửi bởi xuanhoan23112002 on 04-04-2018 - 22:14 trong Phương pháp tọa độ trong mặt phẳng

Cho điểm M(2;1). Lập d đi qua M cắt Ox, Oy tại A, B sao cho khoảng cách từ O đến d là max

d:ax+by-2a-b=0

(a,b khác 0)

Từ đây ta tìm được tọa độ giao điểm của (d) với Ox, Oy theo a, b

Áp dụng hệ thức lượng trong tam giác vuông tính được khoảng cách từ O đến d theo a,b

Tìm max của giá trị.




#712421 $2^n-1$ là số nguyên tố

Đã gửi bởi xuanhoan23112002 on 12-07-2018 - 22:34 trong Số học

Bài này dùng phản chứng thôi.




#705203 Trong một giải đấu bóng đá có 10 đội tham gia theo thể thức mỗi đội đều gặp đ...

Đã gửi bởi xuanhoan23112002 on 08-04-2018 - 08:27 trong Tổ hợp và rời rạc

Ta chứng minh bài toán bằng phản chứng( cả 1 và 2 đều không xảy ra)

Gọi 10 đội bóng là a(i là số tự nhiên và i chạy từ 1 đến 10)

Giả sử a10 là đội bóng có số trận thua nhiều nhất

Khi đó nếu tồn tại giá trị i từ 1 đến 9 mà a10 thang ai thì tất cả cả đội bóng mà a10 thua thì ai cũng thua (vô lí do a10 có số trận thua nhiều nhất)

Suy ra a10 thi đấu với các đội còn lại chỉ có thể hòa hoặc thua

Mà theo gia sư điều kiện 2 không xảy ra nên a10 thua ít nhất 7 đội là aj (j chạy từ 1 đến 7)

Lập luận tương tự như trên với a7 là đội có số trận thua nhiều nhất trong 7 đội trên thì a7 phải thừa ít nhất 4 đội giả sử là: a1, a2, a3, a4.

Lập luận tương tự như trên với a4 là đội có số trận thua nhiều nhất trong 4 đội trên thì a4 phải thừa ít nhất 1 đội giả sử là: a1

Như vậy ta tìm được 4 đội: a1, a4, a7, a10, lập thành 4 đội thỏa mãn điều kiện 1( mâu thuẫn với giả sử)

Do đó giả sử sai. Ta có điều phải chứng minh




#705205 Tìm a? đề hàm số có đạo hàm tại x=-1

Đã gửi bởi xuanhoan23112002 on 08-04-2018 - 08:32 trong Hàm số - Đạo hàm

Ta phải xét tính liên tục của hàm số và đặt điều kiện đạo hàm trái = đạo hàm phải bạn nhé




#709360 Cho $x, y, z > 0$ và $x+y+z= 1$ . Chứng minh

Đã gửi bởi xuanhoan23112002 on 27-05-2018 - 15:33 trong Bất đẳng thức và cực trị

Theo bất đẳng thức Schur ta có:

$(x+y+z)^3+9xyz\geq 4(x+y+z)(xy+yz+zx)$

$\Leftrightarrow 9xyz\geq 4(xy+yz+zx)-1$

$\Leftrightarrow 5xyz+1\geq 4(xy+yz+zx-xyz)$

Theo bất đẳng thức AM-GM ta có:

$xyz\leq \frac{(x+y+z)^3}{27}= \frac{1}{27}$

$\Rightarrow xy+yz+zx-xyz\leq \frac{8}{27}$

Vậy bất đẳng thức được chứng minh.




#704997 tìm giá trị của x để a nguyên

Đã gửi bởi xuanhoan23112002 on 05-04-2018 - 21:04 trong Đại số

PP làm: lấy tử chia mẫu được phần nguyên và phân số rồi sử dụng tính chia hết




#704021 2(ab+bc+ca)+$\frac{1}{ab}+\frac{1...

Đã gửi bởi xuanhoan23112002 on 20-03-2018 - 22:08 trong Bất đẳng thức và cực trị

bạn bị ngược dấu hay sao ấy

nguoc dau cho nao




#709289 Bất đẳng thức trong đề thi vào lớp 10 Nam Định năm 2018

Đã gửi bởi xuanhoan23112002 on 26-05-2018 - 15:38 trong Bất đẳng thức và cực trị

Cho a, b, c là các số thực dương thỏa mãn: $a^2+b^2+c^2+abc=4$. Chứng minh rằng:

$2a+b+c\leq \frac{9}{2}$




#705137 chứng minh

Đã gửi bởi xuanhoan23112002 on 06-04-2018 - 23:24 trong Số học

chứng minh rằng với mọi n$\epsilon$$\mathbb{Z}$ thì $n^{5}$ và n có chữ số tận cùng giống nhau 

n5 - n =n(n-1)(n+1)(n2+1)

Dễ dàng chứng minh n5-n chia hết cho 2

Với n chia hết cho 5, chia 5 dư 1 hoặc 4 thì n5-n chia hết cho 5

Với n chia 5 dư 2 hoặc 3 thì n2+1 chia hết cho 5

Như vậy ta có n5-n chia hết cho 5, n5-n chia hết cho 2 và gcd(5,2)=1

Nên n5-n chia hết cho 10

Hay n5 và n có chữ số tận cùng giống nhau




#706210 Chứng minh MB vuông góc MN khó

Đã gửi bởi xuanhoan23112002 on 17-04-2018 - 20:40 trong Hình học

Bài này có thể sử dụng tích vô hướng của lớp 10




#705432 $\frac{1}{a^2+2b^2+3}+\frac{1}...

Đã gửi bởi xuanhoan23112002 on 10-04-2018 - 21:59 trong Bất đẳng thức và cực trị

Từ giả thiết suy ra abc<=1

a2+2b2+3>=2ab+2b+2>0

$\frac{1}{a2+2b2+3}$<=$\frac{1}{2ab+2b+2}$

Làm tương tự như trên với các phân thức còn lại cùng với abc<=1 ta có điều phải chứng minh




#709817 Đề thi lớp 10 môn Toán vào Trường THPT Chuyên Lam Sơn

Đã gửi bởi xuanhoan23112002 on 03-06-2018 - 10:57 trong Tài liệu - Đề thi

Câu 5: Bất đẳng thức đã cho tương đương với:

$a^2b+b^2c+c^2a+ab^2+bc^2+ca^2> a^3+b^3+c^3+2abc$

$\Leftrightarrow a(b-c)^2+b^2(c+a-b)+c^2(a+b-c)> 0$ (luôn đúng đó a, b, c là độ dài 3 cạnh của 1 tam giác)




#705345 Cần lắm một lời giải thích !

Đã gửi bởi xuanhoan23112002 on 09-04-2018 - 20:48 trong Hình học phẳng

Ý mk hỏi [mod $\pi$] nghĩa là gì

góc định hướng giữa 2 đường thẳng hơn nhau 1 bội của π